Đến nội dung

IHateMath nội dung

Có 282 mục bởi IHateMath (Tìm giới hạn từ 26-05-2020)



Sắp theo                Sắp xếp  

#687365 Tài liệu về cấp và số mũ đúng của 1 số nguyên.

Đã gửi bởi IHateMath on 12-07-2017 - 22:50 trong Tài liệu, chuyên đề, phương pháp về Số học

Nhắc đến số mũ đúng không thể không nhắc đến bổ đề nâng lũy thừa (LTE). Mình xin trích dẫn một link khá đầy đủ về bổ đề này: http://services.arto...XJzaW9uIDMucGRm




#687361 có 8 đội bóng, có bao nhiêu cách chia 8 đội thành 4 cặp đấu

Đã gửi bởi IHateMath on 12-07-2017 - 22:36 trong Xác suất - Thống kê

Có thể tổng quát bài toán lên thành: "Cho $2n$ đội bóng. Có bao nhiêu cách để chia thành $n$ cặp đấu?", và đáp số là $\frac{(2n)!}{2^nn!}$.

Cách đếm như sau: Đánh số lần lượt các đội bóng: $1,1,2,2,\dots ,n,n$ và xếp hai đội cùng số vào một cặp đấu. Vì ta có thể đổi chỗ hai đội có cùng số với nhau, nên có $\frac{(2n)!}{2^n}$ cách để làm như vậy. Tuy nhiên, nếu hoán vị các số $1,2,\dots ,n$ ta lại được một cách xếp trùng với một cách khác. Vậy số cách chia thành $n$ cặp đấu là $\frac{(2n)!}{2^nn!}$.




#687358 Toán Violympic 10, tìm số đại biểu nói được cả ba thứ tiếng

Đã gửi bởi IHateMath on 12-07-2017 - 22:18 trong Mệnh đề - tập hợp

Xét ba tập hợp $A$: Các đại biểu biết tiếng Anh; $B$: Các đại biểu biết tiếng Pháp; $C$: Các đại biểu biết tiếng Nga. Từ đề bài ta thấy rằng:

$$|A\cup B\cup C|=500, |A\cap B|-|A\cap B\cap C|=180, |B\cap C|=170, |A\cap C|=150.$$

Số đại biểu chỉ biết tiếng Anh là $|A|-|A\cap C|-|A\cap B|+|A\cap B\cap C|$,

số đại biểu chỉ biểu tiếng Pháp là $|B|-|B\cap C|-|B\cap A|+|A\cap B\cap C|$,

số đại biểu chỉ biết tiếng Nga là $|C|-|C\cap A|-|C\cap B|+|A\cap B\cap C|$. Vậy số đại biểu chỉ biết một thứ tiếng là:

$$|A|+|B|+|C|-2(|A\cap B|+|B\cap C|+|C\cap A|)+3|A\cap B\cap C|=60.$$

Mà mặt khác,

$$|A|+|B|+|C|-|A\cap B|-|B\cap C|-|C\cap A|+|A\cap B\cap C|=|A\cup B\cup C|=500,$$

cho nên:

$$|A\cap B|+|B\cap C|+|C\cap A|-2|A\cap B\cap C|=440.$$

Suy ra $|A\cap B\cap C|=60$. Đây là đáp số cần tìm.




#687351 Đề luyện tập olympic khối 10 VMF lần 2 tháng 7

Đã gửi bởi IHateMath on 12-07-2017 - 21:31 trong Thi HSG cấp Tỉnh, Thành phố. Olympic 30-4. Đề thi và kiểm tra đội tuyển các cấp.

Bài 5.

Đáp số đúng là $80$ người. 

 

Người $1$ cho đến người $8$: Chỉ không đồng tình với các cải cách lần thứ $1,2,3$.

Người $9$ cho đến người $16$: Chỉ Không đồng tình với các cải cách lần thứ $1,2,4$.

Người $17$ cho đến người $24$: Chỉ Không đồng tình với các cải cách lần thứ $1,2,5$.

Người $25$ cho đến người $32$: Chỉ Không đồng tình với các cải cách lần thứ $1,3,4$.

Người $33$ cho đến người $40$: Chỉ Không đồng tình với các cải cách lần thứ $1,3,5$.

Người $41$ cho đến người $48$: Chỉ Không đồng tình với các cải cách lần thứ $1,4,5$.

Người $49$ cho đến người $56$: Chỉ Không đồng tình với các cải cách lần thứ $2,3,4$.

Người $57$ cho đến người $64$: Chỉ Không đồng tình với các cải cách lần thứ $2,3,5$.

Người $65$ cho đến người $72$: Chỉ Không đồng tình với các cải cách lần thứ $2,4,5$.

Người $73$ cho đến người $80$: Chỉ Không đồng tình với các cải cách lần thứ $3,4,5$.

Người $81$ cho đến người $96$: Đồng tình với tất cả các cải cách.

Rõ ràng, với mỗi cải cách, có đúng $8\cdot 6=48$ người không đồng tình.




#687208 Tìm số nguyên lớn nhất $n$ thỏa $n\equiv 0$ (mod...

Đã gửi bởi IHateMath on 11-07-2017 - 10:52 trong Số học

Nếu ta xét n là giai thừa của một số nguyên và lớn tùy ý thì sao nhỉ?



#687203 MỘT SỐ PHƯƠNG PHÁP GIẢI TOÁN TỔ HỢP THCS

Đã gửi bởi IHateMath on 11-07-2017 - 10:30 trong Toán rời rạc

Mình thì thấy nhiều bài Toán tổ hợp đề bài và lời giải học sinh tiểu học cũng có thể hiểu được. Ví dụ như bài toán con ếch IMO 2016, hay bài đồng xu IMO 2014.



#686955 Có 10 bậc cầu thang

Đã gửi bởi IHateMath on 08-07-2017 - 15:19 trong Toán Tiểu học

Ta tổng quát bài toán lên thành $n$. Gọi $s_n$ là số cách đi. Để ý rằng các cách đi có thể được xếp vào $n$ nhóm dựa vào số bậc trong bước đi cuối cùng:

- Bước $1$ bước duy nhất: $1$ cách duy nhất

- Bước cuối cùng có $n-1$ bậc: $s_1=1$ cách

.

.

.

- Bước cuối cùng có $1$ bậc: $s_n$ cách

Vậy $s_n=s_{n-1}+\dots +s_1+1=s_{n-1}+s_{n-1}=2s_{n-1}=2^{n-1}s_1=2^{n-1}$. 




#686943 đa giác đều có 103 cạnh

Đã gửi bởi IHateMath on 08-07-2017 - 14:24 trong Toán rời rạc

Bài này là bài số 3 của VMO 2014, lời giải có thể tham khảo tại đây: https://diendantoanh...95-h%E1%BB%A3p/




#686917 Chứng minh

Đã gửi bởi IHateMath on 08-07-2017 - 10:26 trong Đại số

Ta có, với mọi $n$ nguyên dương, $n^2<n^2+n+1<(n+1)^2$, do đó luôn luôn không là số chính phương.




#686862 Tìm STN n lớn nhất thỏa mãn 2017! $\vdots$5$^{n...

Đã gửi bởi IHateMath on 07-07-2017 - 21:05 trong Số học

Nếu không nhầm thì:

n=$\left [\frac{2017}{5} \right ]+\left [\frac{2017}{5^{2}} \right ]+\left [\frac{2017}{5^{3}} \right ]+\left [\frac{2017}{5^4} \right ]$

Chỗ này thực chất là công thức Legrendre: https://en.wikipedia...endre's_formula




#686861 Bài toán đếm trong Olympic 27-4 BRVT

Đã gửi bởi IHateMath on 07-07-2017 - 20:58 trong Tổ hợp và rời rạc

Bài toán đã được giải tại đây: https://diendantoanh...án-11/?p=673660




#686806 Cho $n$ là số tự nhiên khác $0$, $a$ là ước ngu...

Đã gửi bởi IHateMath on 07-07-2017 - 16:12 trong Đại số

Chỉ đơn thuần là nhân hai vế với $\left( \frac{k}{n}\right) ^2$ thôi, còn $k$ đã là số nguyên dương thì $k^2+2k$ cũng là số nguyên dương.




#686787 Cấp số cộng và phân hoạch tập $Z^+$

Đã gửi bởi IHateMath on 07-07-2017 - 15:02 trong Số học

Cho số nguyên tố $p\geq 3$ và dãy số nguyên phân biệt $\{ a_i\}_{i=1}^p$. Chứng minh rằng nếu tập số nguyên dương $Z^+$ có thể phân hoạch thành các tập $A_1,A_2,\dots ,A_p$ sao cho với các $i\in\{ 1,2,\dots ,p\}$ thì các tập $A_i+a_i=\{x+a_i|x\in A_i\}$ là đồng nhất thì dãy $\{ a_i\}$ phải lập thành một cấp số cộng.




#686718 Tô màu được nhiều nhất bao nhiêu ô trên bàn cờ?

Đã gửi bởi IHateMath on 06-07-2017 - 17:59 trong Tổ hợp và rời rạc

Kí hiệu $f(n)$ thay cho số lớn nhất các ô vuông được tô màu.

Đáp số: $f(n)=\frac{n^2}{2}$ nếu $n$ chẵn và $f(n)=\frac{n(n+1)}{2}$ nếu $n$ lẻ.

Chứng minh

Trường hợp $n$ chẵn là hiển nhiên. Ta chỉ xét trường hợp $n$ lẻ. Ta sẽ chứng minh bằng quy nạp. Trong trường hợp cơ sở $n=1$ hiển nhiên $f(1)=1$, đúng. Giả sử ta đã chứng minh được rằng $f(2k-1)=(2k-1)k$ với $k$ nguyên dương nào đó. Trước hết ta sẽ chứng minh

$$f(2k+1)\leq(2k+1)(k+1)\quad (1)$$ 

Thật vậy ta cắt bảng $(2k+1)\times (2k+1)$ thành các bảng con như sau: một bảng $(2k-1)\times (2k-1)$ sao cho nó có chung một đỉnh với bảng lớn và phần còn lại cắt thành các bảng con $2\times 2$ chừa ra một hình L - Tetramino (là hình gồm 4 ô vuông xếp với nhau tạo thành hình chữ "L").

Theo giả thuyết quy nạp, trên bảng con $(2k-1)\times (2k-1)$ tô được nhiều nhất $(2k-1)k$ ô vuông còn trên hình L - Tetramino tô được nhiều nhất $3$ ô (điều này khá hiển nhiên), các bảng con $2\times 2$ (có đúng $2k-1$ bảng như vậy) thì tô được nhiều nhất $2$ ô, nên trên bảng ban đầu ta tô được nhiều nhất:

$$(2k-1)k+2(2k-1)+3=(2k+1)(k+1).$$

Điều này chứng tỏ $(1)$ đúng.

Giờ ta sẽ chỉ ra rằng dấu "=" trong $(1)$ là có thể xảy ra. Phần này thì khá đơn giản, chỉ cần tô xen kẽ các hàng là xong.

Vậy công thức đã cho cũng đúng với $n=2k+1$. Chứng minh kết thúc.

Bình luận: Trong trường hợp $n=5$ bài toán từng xuất hiện trên tạp chí THTT.




#685292 CMR:a/(a+b) + b/(b+c) + c/(c+a) <= 3/2

Đã gửi bởi IHateMath on 21-06-2017 - 18:56 trong Bất đẳng thức và cực trị

Sau không liên quan vì đề sai.

Bạn chưa đọc kỹ đề à? Dề cho: $\sum{\frac{a}{a+b}}$, còn bất đẳng thức Nesbitt lại là $\sum{\frac{a}{b+c}}$.




#685290 CMR:a/(a+b) + b/(b+c) + c/(c+a) <= 3/2

Đã gửi bởi IHateMath on 21-06-2017 - 18:52 trong Bất đẳng thức và cực trị

Không thể có bđt đó vì  ta có: $\sum \frac{a}{b+c}\geq \frac{3}{2}$ (Đây chính là bđt nesbit)

Thật chả liên quan!




#685093 Tìm nghiệm nguyên của phương $x^{y}+1=z$

Đã gửi bởi IHateMath on 20-06-2017 - 00:25 trong Đại số

Nếu không có thêm điều kiện gì khác thì rõ ràng phương trình đã cho có vô số bộ nghiệm nguyên $(x,y,z)$: chẳng hạn như $(2,t,2^t+1)$ với $t\in\mathbb{N}$.




#684860 Cho 3 số dương $a,b,c$ thoả mãn: $\sum \frac{1...

Đã gửi bởi IHateMath on 18-06-2017 - 00:49 trong Bất đẳng thức - Cực trị

Bài này tương tự bài toán $\text{A2 IMO 2009 Shortlist}$, chỉ thay điều kiện $\frac{1}{a}+\frac{1}{b}+\frac{1}{c}=a+b+c$. Lời giải cũng tương tự như bài toán nói trên.




#684859 ĐỀ THI TUYỂN SINH VÀO LỚP 10 MÔN TOÁN CHUYÊN TRƯỜNG THPT CHUYÊN LÊ KHIẾT

Đã gửi bởi IHateMath on 18-06-2017 - 00:36 trong Tài liệu - Đề thi

Chứng minh giúp tam giác MAB đồng dạng NAC tý các cao thủ

Dễ thấy $\Delta ABD\sim\Delta ACE$, mà $AM,\, AN$ lần lượt là trung tuyến của $\Delta ABD,\, \Delta ACE$ nên $\Delta ABM\sim\Delta ACN$.




#684673 $(a+b+c)^3\geq 6\sqrt{3}(a-b)(b-c)(c-a)$

Đã gửi bởi IHateMath on 16-06-2017 - 10:55 trong Bất đẳng thức và cực trị

Không mất tính tổng quát, giả sử $a\geq b,\, a\geq c$. Ta xét hai trường hợp sau:

$\bullet$ Trường hợp $1$: $b\geq c$. Khi đó $(a-b)(b-c)(c-a)\leq0$, trong khi $a+b+c\geq 0$. Bất đẳng thức đa cho đúng. Dấu "=" xảy ra khi $a=b=c=0$.

$\bullet$ Trường hợp $2$: $b<c$. Khi đó, do $b\geq 0$ nên ta có $$(a+b+c)^3\geq (a+c)^3,\, (a-b)(b-c)(c-a)=(a-b)(c-b)(a-c)\leq ac(a-c).$$

Như vậy ta chỉ còn cần phải chứng minh

$$(a+c)^3\geq 6\sqrt{3} ac(a-c).$$

Đặt $t=\frac{a}{c}$ thì $t\geq 1$ và bất đẳng thức trên trở thành

$$(t+1)^3\geq 6\sqrt{3} t(t-1)\iff (t-2-\sqrt{3})^2(t+7-4\sqrt{3})\geq 0,$$

hiển nhiên đúng. Dấu "=" xảy ra khi $b=0,\, a=(2+\sqrt{3})c\, (c>0)$.

 




#684581 Không hiểu một vài chỗ trong bài chứng minh sự tồn tại vô hạn các số nguyên tố.

Đã gửi bởi IHateMath on 14-06-2017 - 23:55 trong Đại số

Cảm ơn bạn nhiều nhé.

 

Mình có 1 bài này nữa cũng thắc mắc, mong bạn giải đáp thêm:

 

Cho số nguyên $n$ là hợp số, $n > 1$. Chứng minh rằng $n$ có ước nguyên tố $p \leqslant \sqrt{n}$.

 

Cách giải (của sách):

 

Do $n$ là hợp số nên $n$ có thể viết dưới dạng $n = a.b$ với $a, b \in N, a > 1, b > 1$. Bây giờ nếu cả $a > \sqrt{n}$ và $b > \sqrt{n}$ thì $ab > \sqrt{n}.\sqrt{n} = n$, mâu thuẫn. Do đó phải có $a \leqslant \sqrt{n}$ hoặc $b \leqslant \sqrt{n}$, và do đó $n$ có ước nguyên tố $p \leqslant \sqrt{n}$. Bài toán được chứng minh.

 

1. Vì sao $n$ là hợp số nên $n$ có thể viết được dưới dạng $n = a.b$ với $a, b \in N, a > 1, b > 1$? (giả sử $n = 4$ thì trường hợp $4 = 4.1$ cũng có thể xảy ra mà?)

2. Vì sao có $a \leqslant \sqrt{n}$ hoặc $b \leqslant \sqrt{n}$ mà $n$ lại có ước nguyên tố $p \leqslant \sqrt{n}$?

 

Mình cảm ơn.

Em thấy lời giải này đâu có sai sót nhỉ?

Để mình giải thích cho bạn.

1. $n$ là hợp số thì $n$ phải có ước số khác $1$ và $n$, ta gọi nó là $d$ đi, thì chả phải $\frac{n}{d}$ và $d$ đều lớn hơn $1$ hay sao? Bạn đưa ra ví dụ $4=4\cdot 1$ để làm gì nhỉ, vì rõ ràng $4=2\cdot 2$.

2. Giả sử $a\leq \sqrt{n}$ thì $a$ chắc chắn sẽ có ước nguyên tố và ta gọi nó là $p$, thì $n$ chia hết cho $p$, mà $p\leq a\leq \sqrt{n}$ nên sẽ có điều cần chứng minh.




#684576 Bài toán lát gạch $4\times 3$

Đã gửi bởi IHateMath on 14-06-2017 - 23:23 trong Tổ hợp và rời rạc

Lát hay lát kín ạ 

 

lát thừa ra được không 

"Lát" ở đây là lát kín, đồng thời không có viên nào chờm lên nhau hay dư ra ngoài.




#684149 Đề tuyển sinh vào lớp 10 THPT chuyên Lê Quý Đôn - Quảng Trị năm học 2017-2018

Đã gửi bởi IHateMath on 12-06-2017 - 00:03 trong Tài liệu - Đề thi

Câu 6.

 

Ta quy về chứng minh Bài Toán sau: Cho tam giác $ABC$ nhọn. $M$ là trung điểm của $BC$. Đường cao $BH , CK$ lần lượt cắt đường thẳng vuông góc với $AM$ tại $E , F$. Chứng minh rằng: $AE=AF$

 

Mình có cách giải khác khác cho bài toán này. Ta cần chứng minh $$\frac{AK}{\cos\angle BAF}=\frac{AH}{\cos\angle CAE}\iff \frac{\cos\angle BAF}{\cos\angle CAE}=\frac{AK}{AH}.$$

Do các góc $\angle BAF,\, \angle BAM$ phụ nhau nên cosin góc này bằng sin góc kia. Tương tự với các góc $\angle CAE,\, \angle CAM$.

Vậy điều cần chứng minh trở thành $$\frac{\sin\angle BAM}{\sin\angle CAM}=\frac{AK}{AH}.$$

Mặt khác, theo định lý Sin, diện tích tam giác $BAM$ bằng $\frac{1}{2}\cdot AB\cdot AM\cdot\sin \angle BAM$, diện tích tam giác $CAM$ bằng $\frac{1}{2}\cdot AC\cdot AM\cdot\sin \angle CAM$. Hơn nữa hai tam giác này có diện tích bằng nhau nên

$$AB\cdot\sin\angle BAM=AC\cdot\sin\angle CAM\Rightarrow \frac{\sin\angle BAM}{\sin\angle CAM}=\frac{AC}{AB}.$$

Vậy ta chỉ cần phải chứng minh $AH\cdot AC=AK\cdot AB$, mà điều này lại đúng do tứ giác $BKHC$ nội tiếp. 

P/s: $\downarrow$ - Cảm ơn bạn, mình post bài này lúc nửa đêm :-D




#684087 Đề thi vào 10 chuyên tỉnh Hậu Giang 2017-2018

Đã gửi bởi IHateMath on 11-06-2017 - 15:42 trong Tài liệu - Đề thi

Câu $1.3$.

 

Ta có $\frac{p}{x}+\frac{q}{y}=1\iff pq=(y-q)(x-p)$. Suy ra $(x,y)\in\{(p+1,pq+q),(pq+p,q+1),(2p,2q),(p+q,p+q)\}$.




#684085 Đề thi vào 10 chuyên tỉnh Hậu Giang 2017-2018

Đã gửi bởi IHateMath on 11-06-2017 - 15:14 trong Tài liệu - Đề thi

Câu 1a)

Ta có: $(a+1)(b+1)\geq 64$

$\Leftrightarrow ab+a+b+1\geq 64$

$\Leftrightarrow ab+a+b\geq 63$(1)

Mặt khác,

$(a+b)^{2}\geq 4ab(dễ dàng chứng minh)$

$\Rightarrow \frac{1}{4}(a+b)^{2}\geq ab$(2)

$(1)\wedge(2)\Rightarrow \frac{1}{4}(a+b)^{2}+a+b\geq ab+a+b\geq 63$

$\Rightarrow \frac{1}{4}(a+b)^{2}+a+b-63\geq 0$

Đặt a+b=x(x>0),bpt trở thành

$\frac{1}{4}x^{2}+x-63\geq 0$

Giải bpt trên , ta được:

$\left\{\begin{matrix}X\leq -18(L) & \\X\geq 14(N) & \end{matrix}\right.$

$\Rightarrow Min(a+b)=14.Dấu bằng xảy ra \Leftrightarrow$

$\left\{\begin{matrix}(a+1)(b+1)=64 & \\a=b & \end{matrix}\right.$

$\Leftrightarrow a=b=7$

Vậy Min(a+b)=14 <=>a=b=7

Đâu cần phức tạp thế nhỉ?

Ta có $a+b+2=(a+1)+(b+1)\geq 2\sqrt{(a+1)(b+1)}=16\Rightarrow a+b\geq 14$. Dấu "=" xảy ra khi và chỉ khi $a=b=7$.